Torts Online Questions

Lakukan tugas rumah & ujian kamu dengan baik sekarang menggunakan Quizwiz!

Alice sues Bill alleging negligence. Bill defends, claiming that Alice's negligence also contributed to her injuries. At trial, the factfinder assigns 40% responsibility to Alice and 60% responsibility to Bill. The factfinder also finds that Alice's damages are $100,000. In a pure comparative fault jurisdiction, how much is Alice entitled to recover? A) Alice is entitled to recover $60,000 B) Alice is entitled to recover $100,000 C) Alice is not entitled to recover anything because she was partially responsible D) Alice is entitled to recover $40,000

A) Alice is entitled to recover $60,000

City is 100 square miles and contains almost 60,000 trees along its streets. Barney is driving on a street during a windstorm when a tree on City property adjacent to the street falls on his car. Barney's car is damaged, and he suffers physical injuries. City had planted the tree some 15 years previously, and upon visual inspection by City workers, the tree appeared to be quite healthy. City has trained staff, which inspects each city tree annually, and employees of other City departments are under strict instruction to keep their eyes open for trees in a hazardous condition as they perform their regular duties. However, at the time of the storm, the tree was rotting due to rare disease from its inside out, and the weakened state of this particular tree caused it to fall under the pressure of the storm. Even when a tree does go bad as this one did, the chances of it suddenly falling in circumstances likely to produce serious property or personal injury are quite small. City had no actual knowledge of this tree's condition. Only a much more costly inspection program could have successfully detected the rotting tree that fell on Barney. In a negligence action brought by Barney against City, will Barney likely be able to hold city liable in negligence? a) Barney will likely not be able to hold City liable in negligence because the cost to City of detecting the dangerous condition of the tree was prohibitive b) Barney will likely be able to hold City liable in negligence for failing to enact the more costly inspection program if it could have possibly prevented injuries such as the ones sustained by Barney c) Barney will likely not be able to hold City liable in negligence because the tree fell only due to the windstorm d) Barney will likely be able to hold City responsible in negligence because Barney was injured as a result of the tree's dangerous condition

A) Barney will likely not be able to hold City liable in negligence because the cost to City of detecting the dangerous condition of the tree was prohibitive

In a negligence action brought by Plaintiff Betty against Defendant Alice and Defendant Carl, the jury apportions liability based on the negligence of the parties as follows: Betty is 20% at fault; Alice is 60% at fault; and Carl is 20% at fault. The jury also determines that Plaintiff Betty sustained $100,000 in compensatory damages. The parties are in a modified "not greater than" comparative fault jurisdiction that provides that defendants are severally liable, where the plaintiff's fault is compared to the combined fault of all defendants. What amount may Betty recover from Alice or Carl? A) Betty may recover 60% of $100,000 from Alice and 20% of $100,000 from Carl B) Betty may recover 80% of $100,000 from either Alice or Carl or a combination of both C) Betty may recover 60% of $80,000 from Alice and 20% of $80,000 from Carl D) Betty may recover 50% of $100,000 from Carl and 50% of $100,000 from Alice

A) Betty may recover 60% of $100,000 from Alice and 20% of $100,000 from Carl

In a negligence action brought by Plaintiff Betty against Defendant Alice and Defendant Carl, the jury apportions liability based on the negligence of the parties as follows: Betty is 20% at fault; Alice is 60% at fault; and Carl is 20% at fault. The jury also determines that Plaintiff Betty sustained $100,000 in compensatory damages. In a pure comparative fault jurisdiction that provides that defendants are severally liable, what amount, if any, may Betty recover? A) Betty may recover up to 60% of $100,000 from Alice and up to 20% of $100,000 from Carl B) Betty may recover up to 60% of $80,000 from Alice and up to 20% of $80,000 from Carl C) Betty may recover 60% of $80,000 from Alice or Carl or a combination of both D) Betty may not recover at all

A) Betty may recover up to 60% of $100,000 from Alice and up to 20% of $100,000 from Carl

Drug Co. negligently manufactures and sells a defective contraceptive pill, which fails to prevent Yolanda's pregnancy. Yolanda gives birth to Child A, who grows up and gives birth to Child B. Child B grows up to be a motorcycle enthusiast, who negligently causes a motorcycle accident that injures Charles. Charles seeks to recover from Drug Co. in a negligence action, alleging that Drug Co.'s negligence was a factual cause of Charles's injury. Will Charles's negligence action likely succeed or fail? A) Charles is correct that Drug Co. is a factual cause of Charles's injury, but his negligence action will likely fail for lack of proximate cause B) Charles is incorrect that Drug Co. is a factual cause of Charles's injury and therefore cannot be subject to liability in negligence to Charles C) Charles is correct that Drug Co. is a factual cause of Charles's injury, and therefore his negligence action against Drug Co. will likely succeed D) Charles is incorrect that Drug Co. is a factual cause of Charles's injury, but his negligence action against Drug Co. may still succeed

A) Charles is correct that Drug Co. is a factual cause of Charles's injury, but his negligence action will likely fail for lack of proximate cause

A state statute requires that the operator of a truck that becomes disabled on a highway promptly put out a warning sign at least 100 feet behind the truck. When a deflated tire disables Chuck's truck, he promptly places out a warning sign right next to the truck rather than at the 100-foot distance. Alice, approaching Chuck's truck from behind, does not see Chuck's warning sign until it is too late for her to stop. Her car strikes the rear of Chuck's truck, and she is injured in the collision. Alice sues Chuck in negligence, and the evidence at trial shows that Alice would have been able to stop in time had the warning sign been set at the 100-foot distance as required by statute. What effect, if any, does Chuck's violation of the statute have on his liability for negligence? A) Chuck's violation of the statute may be used to establish his negligence, because the basic purpose of the statute is to prevent accidents of this type and Alice is in the group of persons the statute is designed to protect B) Chuck's violation of the statute is evidence of res ipsa loquitur because the basic purpose of the statute is to prevent accidents of this type and Alice is in the group of persons the statute is designed to protect C) Chuck's violation of the statute has no effect on his liability for negligence because the statute cannot be used to establish the standard of care in Alice's negligence action against Chuck because the statute was not designed expressly to provide a private right of action in negligence D) Chuck's violation of the statute has no effect on his liability for negligence because the standard of care in negligence must be established by proving what the reasonable prudent person in the same or similar circumstances would have done

A) Chuck's violation of the statute may be used to establish his negligence, because the basic purpose of the statute is to prevent accidents of this type and Alice is in the group of persons the statute is designed to protect

About two years ago, Client was a patient of Ophthalmologist. Client alleges that during the time that he was Ophthalmologist's patient, Ophthalmologist improperly performed LASIK surgery on Client's eyes. The LASIK surgery was supposed to correct Client's poor vision. Instead, the surgery was performed skillfully, but resulted in blindness in both eyes, a risk about which Client was never informed and a reasonable person would not have known. If Client had known of the risk of blindness, he would not have opted for the surgery. Client hires Attorney Matthew to sue Ophthalmologist for medical malpractice. Attorney Matthew fails to file the complaint to initiate the cause of action against Ophthalmologist within the applicable statute of limitations. As a result, the court dismisses Client's case against Ophthalmologist. Client then hires Attorney Becky to represent him in a cause of action against Attorney Matthew for legal malpractice. In the legal malpractice action brought by Client against Attorney Matthew, how will the professional standard of care owed by Attorney Matthew need to be established? A) Client will have to establish, by introducing expert testimony, that Attorney Matthew failed to perform as the reasonable prudent attorney in handling Client's medical malpractice action against Ophthalmologist B) Client will be able to establish that Attorney Matthew failed to perform as the reasonable prudent attorney in handling Client's medical malpractice action without expert testimony regarding the standard of care C) Client will be able to establish the standard of care in the legal malpractice action against Attorney Matthew by applying a rule of law D) The factfinder will determine the standard of care in this legal malpractice action without evidentiary guidance

A) Client will have to establish, by introducing expert testimony, that Attorney Matthew failed to perform as the reasonable prudent attorney in handling Client's medical malpractice action against Ophthalmologist

` Defendant, driving a car, approaches Plaintiff's car, and properly steps on the brakes. For reasons unrelated to any negligence on Defendant's part, her brakes fail to function. Much to her surprise, Defendant's car continues forward, so she quickly pumps the brakes again, but to no avail. Defendant's car careens into Plaintiff's car, injuring Plaintiff. Upon reflecting calmly on Defendant's brake-failing situation, a better choice for Defendant would have been for her to turn her car quickly to the right. In Plaintiff's lawsuit against Defendant in which he seeks to recover from her in negligence, will Defendant likely be held liable in negligence? A) Defendant will likely not be held liable for negligence because in light of her emergency circumstances, she behaved as a reasonable prudent person B) Defendant will likely be held liable in negligence because she should have chosen more wisely by turning the car quickly to the right C) Defendant will likely be held liable in negligence if she caused Plaintiff's injuries D) Defendant will likely not be held liable in negligence because she was reacting to an emergency situation

A) Defendant will likely not be held liable for negligence because in light of her emergency circumstances, she behaved as a reasonable prudent person

The mascot for the defendant Professional Baseball Team (PBT) injures Freddy Fan when he throws a hot dog into the stands. The jurisdiction's highest court has held in the past that a professional baseball team owes no duty to a patron who is injured by a food item or souvenir tossed into the stands as a promotional stunt by a team employee. If Fan sues PBT alleging negligence, the trial court judge should: A) Dismiss the case as a matter of law because PBT owes Fan no duty of care B) Grant a directed verdict in favor of PBT because a reasonable jury could not conclude that the actions of its mascot violated a duty of care owed to Fan C) Allow the case to be heard by the jury, but instruct the jury that no duty of care is owed by PBT to Fan D) Allow the jury to decide whether PBT's mascot acted negligently

A) Dismiss the case as a matter of law because PBT owes Fan no duty of care

Rosie and Vincent do not know each other and were camping independently in a heavily forested campground. Each one had a campfire, and each one negligently failed to ensure that the fire was extinguished upon retiring for the night. Due to unusually dry forest conditions and a stiff wind, both campfires escaped their sites. The two fires, burning out of control, eventually joined together and engulfed Company's hunting lodge, destroying it. Either fire alone would have destroyed the lodge. In a lawsuit brought by Company against both Rosie and Vincent alleging negligence, which of the following statements is correct regarding factual causation? A) Each of Rosie's and Vincent's negligence is a factual cause of the destruction of Company's hunting lodge B) Neither Rosie's nor Vincent's negligence is a factual cause of the destruction of Company's hunting lodge C) Rosie's negligence is the sole factual cause of the destruction of Company's hunting lodge D) Vincent's negligence is the sole factual cause of the destruction of Company's hunting lodge

A) Each of Rosie's and Vincent's negligence is a factual cause of the destruction of Company's hunting lodge

Charles is a guest in Heights Hotel. The bathroom in the hotel includes a shower, which is protected by a sliding door made of ordinary glass. While taking a shower, Charles trips and falls on the glass door, causing it to shatter. The shards of glass cut Charles causing serious injuries. It is a standard practice among hotels to use shatterproof, safety glass rather than ordinary glass at shower enclosures. In a negligence action brought by Charles against Heights Hotel, what effect, if any, will Hotel's departure from the industry custom have on Hotel's negligence? A) Heights Hotel's departure from this custom is evidence of its negligence B) Heights Hotel's departure from this custom is conclusive proof of its negligence C) Evidence of Heights Hotel's departure from this industry custom is legally irrelevant to the issue of its negligence D) Despite evidence of that Heights Hotel disregarded custom, Heights Hotel may not be subject to liability in negligence because Charles tripped and fell, causing his injuries

A) Heights Hotel's departure from this custom is evidence of its negligence

Hunter finishes his day working in the field and stops at a friend's house on his walk home from work. His friend's nine-year-old daughter, Katie, greets Hunter, who hands his loaded shotgun to her as he enters the house. The shotgun is neither especially heavy nor unwieldy. Nonetheless, Katie drops the shotgun, which lands on her toe, breaking it. In a negligence action against Hunter for Katie's broken toe, may Hunter be held liable in negligence? A) Hunter may not be held liable in negligence for Katie's broken toe because the risk that makes Hunter negligent is that Katie might shoot something or some person with the gun, not that she would drop it on her toe and hurt herself B) Hunter may be held liable in negligence for Katie's broken toe because he negligently provided her with the shotgun C) Hunter may not be held liable because Hunter's act of giving Katie the gun is not a factual cause of Katie's broken toe D) Hunter may not be held liable for Katie's broken toe if the manner in which she ended up injuring herself was unforeseeable.

A) Hunter may not be held liable in negligence for Katie's broken toe because the risk that makes Hunter negligent is that Katie might shoot something or some person with the gun, not that she would drop it on her toe and hurt herself

Connie is carelessly changing lanes in her car when her car is struck by Bill who is reaching for his phone, which has fallen under the passenger side seat while he was driving. Adrian, who is in the vicinity on her motorcycle waiting patiently at the intersection for the light to turn green, is struck by Connie's car and suffers serious physical injury as well as damage to her bike. She sues both Connie and Bill for their negligence. The jury determines that Adrian's damages are $100,000 and concludes that Bill and Connie are both liable. It assigns 30% responsibility to Bill, and 70% responsibility to Connie. In a jurisdiction that subjects defendants to joint and several liability, what amount, if any, is Adrian entitled to recover from Bill and/or Connie? A) If he chooses, Adrian may recover $100,000 from Bill alone B) Adrian is entitled to recover only $30,000 from Bill and only $70,000 from Connie for a total of $100,000 C) Adrian is entitled to recover $100,000 from both Bill and Connie for a total of $200,000 D) If he chooses, Adrian may recover up to $30,000 from Connie and $70,000 from Bill

A) If he chooses, Adrian may recover $100,000 from Bill alone

Plaintiff Pammy, a passenger in a car driven by Edgar, is injured in an accident resulting from the negligence of both Edgar and Mitch, the driver of the car that collides with Edgar's car. Pammy sues both Edgar and Mitch. Which result is most likely in a jurisdiction still following the principle that traditional contributory negligence is a total bar to recovery? A) Pammy will be able to recover from both Edgar and Mitch B) Pammy will be able to recover from the driver of the car in which she was riding, Edgar. However, she will not be able to recover from Mitch, because Edgar's negligence will be imputed to her as contributory negligence and will bar her recovery C) Pammy will be able to recover from Mitch, but not from Edgar because of the automobile guest statute D) Pammy will not be able to recover from Mitch because of imputed contributory negligence. Nor will she be able to recover from Edgar because of the automobile guest statute

A) Pammy will be able to recover from both Edgar and Mitch

Which of the following statements about factual and proximate causation is correct? A) Proximate causation determinations in negligence actions establish whether to shield a defendant from liability even though the defendant's breach of the standard of care was a factual cause of the plaintiff's harm B) Proximate causation determinations are unique to negligence actions and are not made in intentional torts or strict liability actions C) Legal causation and factual causation are synonymous terms D) Factual causation and proximate causation are synonymous terms

A) Proximate causation determinations in negligence actions establish whether to shield a defendant from liability even though the defendant's breach of the standard of care was a factual cause of the plaintiff's harm

Shopper is picking up a few items at Grocery Store on her way home from work. As she enters the produce section of Grocery Store, she sees Plaintiff lying on the floor, holding her obviously injured hip. Right next to Plaintiff, Shopper observes a dry, gritty, almost entirely black, squishy banana lying on the floor. In Plaintiff's negligence action against Grocery Store alleging Grocery Store's negligence, Shopper testifies to the condition of the banana when she saw it as it lay next to Plaintiff who was on the floor as well. What is the best characterization of Shopper's testimony presented in the trial brought by Plaintiff against Grocery Store for its alleged negligence? A) Shopper's testimony is circumstantial evidence of Grocery Store's negligence and requires the factfinder to draw an inference regarding Grocery Store's breach of the standard of care B) Shopper's testimony is direct evidence of Grocery Store's negligence and requires the factfinder to draw an inference regarding Grocery Store's breach of the standard of care C) Shopper's testimony is circumstantial evidence and cannot be used to establish Grocery Store's breach of the standard of care D) Shopper's testimony is direct evidence of Grocery Store's negligence and cannot be used to establish Grocery Store's breach of the standard of care

A) Shopper's testimony is circumstantial evidence of Grocery Store's negligence and requires the factfinder to draw an inference regarding Grocery Store's breach of the standard of care

Small Corp. is vicariously liable for a tort committed by one of its employees, Chaz, and pays a judgment to the plaintiff of $100,000. It then proceeds to file suit in a court in an appropriate jurisdiction seeking to recover the entire $100,000 from Chaz. In this jurisdiction, when independent tortfeasors contribute to an indivisible harm, the courts apply several liability. What result and why? A) Small Corp. will recover the entire $100,000 from Chaz in an action seeking indemnity B) Small Corp. will recover nothing because it was liable to plaintiff under the principle of vicarious liability C) Small Corp. will be able to recover a portion of the $100,000 judgment it has paid to plaintiff in a contribution action D) Small Corp. will be able to recover $100,000 from Chaz only if his employment contract granted the employer a right of indemnity for any losses Chaz caused during his employment

A) Small Corp. will recover the entire $100,000 from Chaz in an action seeking indemnity.

George, a concessionaire at a sports stadium, negligently collided with Fanny, a fan attending the game, knocking her to the ground. Fanny had an asymptomatic herniated disc in her back that results in her suffering serious back injury and pain as a result of George's collision with her. In a negligence action brought by Fanny against George, may George be subject to liability for Fanny's injuries? A) That Fanny could be injured was foreseeable from George's negligence and therefore George can be subject to liability B) George cannot be held liable for Fanny's injuries because he could not have foreseen that she was suffering from an asymptomatic herniated disc C) George may be subject to liability for the collision itself, but not for the extensive back injuries because the extensive back injuries were not foreseeable D) George may only be held liable to Fanny for injuries that he intended to cause

A) That Fanny could be injured was foreseeable from George's negligence and therefore George can be subject to liability

Alice was exposed to two different solvents while working in a laboratory. Each solvent contained a toxic chemical. One contained Chemical B; the other contained Chemical C. After developing the disease decolis several years later, Alice sues the manufacturers of each solvent, claiming that the manufacturers were negligent for including a toxic chemical in their solvents. Alice's evidence, presented by competent expert testimony based on valid scientific evidence reveals that the mechanism by which decolis develops is different for Chemical C exposure and for Chemical B exposure and that exposure to one or the other but not both is the most likely explanation for Alice's decolis. However, Alice cannot prove whether Chemical B or Chemical C caused her decolis. After presentation of her case, the defendants (manufacturers of Chemical B and manufacturer of Chemical C) both move for summary judgment on the issue of factual causation. How should the court rule on the defendants' requests for summary judgment? A) The court should deny the motion for summary judgment and rule that the burden of proof on agent-disease factual causation is shifted to the manufacturers of Chemical B and Chemical C B) The court should grant the motion for summary judgment because Alice has failed to establish which defendant is the factual cause of her decolis C) The court should deny the motion for summary judgment because the defendants in negligence action always have the burden of proof on the issue of causation D) The court should grant the motion for summary judgment so that Alice can engage in further discovery on the issue of causation

A) The court should deny the motion for summary judgment and rule that the burden of proof on agent-disease factual causation is shifted to the manufacturers of Chemical B and Chemical C

Defendant Doctor treated Plaintiff Patient for a minor medical problem. Defendant, in an attempt to reduce his medical malpractice premiums, asked Plaintiff to sign a "Disclaimer of Liability" that provided that Plaintiff waived all her rights to seek compensation for injuries resulting from the negligence of the Defendant, but allowed her to recover any damages caused by intentional, wanton or reckless conduct on the part of the Defendant. In exchange, Plaintiff's co-payment for medical services was waived. Defendant carefully explained the disclaimer of liability orally and in a written explanation that clearly stated that the patient would be excluding liability for negligence. She also readily indicated that Plaintiff was not required to sign the disclaimer of liability. Plaintiff signed. As a result of the medical treatment, she sustained severe harm. She now sues Defendant for medical negligence. What is the most likely effect of the disclaimer on the lawsuit? A) The disclaimer will have no effect on the Plaintiff's recovery because the provision of medical services involves the public interest. B) The disclaimer will have no effect on the Plaintiff's recovery because a plaintiff can never disclaim liability for negligence C) The disclaimer will be effective because the disclaimer was in writing and explicitly mentioned negligence D) The disclaimer will be effective because Plaintiff's agreement to the disclaimer did not result from a gross disparity of bargaining power and the disclaimer does not waive liability for intentional, wanton or reckless conduct

A) The disclaimer will have no effect on the Plaintiff's recovery because the provision of medical services involves the public interest.

Toddler, a two-year-old child suffering from a bacterial infection, received a routine childhood vaccination with a vaccine manufactured by Vaco. Shortly after receiving the vaccine, Toddler spiked a very high fever, went into respiratory arrest, and died. Toddler's estate sues Vaco in negligence. At trial, Toddler's estate provides competent evidence that Toddler died due to a combination of the infection and the vaccine and, if either toddler had not had an infection or if Toddler had not been vaccinated, death would not have ensued. Regarding the factual causation element of the negligence action, has Toddler's estate satisfied its burden of production to prove that the vaccine was a factual cause of Toddler's death? A) Toddler's estate has satisfied its burden of production to show that the vaccine was a factual cause of Toddler's death B) Toddler's estate has not satisfied its burden of production to show that the vaccine was a factual cause of Toddler's death C) Toddler's estate cannot satisfy its burden of production to show that the vaccine was a factual cause of Toddler's death because the infection contributed to Toddler's death D) Toddler's estate cannot satisfy its burden of production to show that the vaccine was a factual cause of Toddler's death because the infection was the sole factual cause of Toddler's death

A) Toddler's estate has satisfied its burden of production to show that the vaccine was a factual cause of Toddler's death

Alice sues Bill alleging negligence. Bill defends, claiming that Alice's negligence also contributed to her injuries. At trial, the factfinder assigns 50% responsibility to Alice and 50% responsibility to Bill. The factfinder also finds that Alice's damages are $100,000. In a modified comparative fault (51% or "not greater than") jurisdiction, what amount, if any, is Alice entitled to recover? A) Alice is entitled to recover $50,000 from Bill because Bill only has to pay for his percentage of the judgment attributable to his fault B) Alice is entitled to recover $50,000 from Bill because her recoverable damages are her damages reduced by the amount of fault attributable to her C) Alice is entitled to recover $100,000 from Bill D) Alice is barred from recovering from Bill because she was partially at fault

B) Alice is entitled to recover $50,000 from Bill because her recoverable damages are her damages reduced by the amount of fault attributable to her

Alice sues Bill alleging negligence. Bill defends, claiming that Alice's negligence also contributed to her injuries. At trial, the factfinder assigns 50% responsibility to both Alice and Bill. The factfinder also finds that Alice's damages are $100,000. In this pure comparative fault jurisdiction, what amount, if any, is Alice entitled to recover? A) Alice is entitled to recover $50,000 from Bill because Bill only has to pay for his percentage of the judgment attributable to his fault B) Alice is entitled to recover $50,000 from Bill because her recoverable damages are her damages reduced by the amount of fault attributable to her C) Alice is entitled to recover $100,000 from Bill D) Alice is completely barred from recovering from Bill because she was partially at fault

B) Alice is entitled to recover $50,000 from Bill because her recoverable damages are her damages reduced by the amount of fault attributable to her

About two years ago, Client was a patient of Ophthalmologist. Client alleges that during the time that he was Ophthalmologist's patient, Ophthalmologist improperly performed LASIK surgery on Client's eyes. The LASIK surgery was supposed to correct Client's poor vision. Instead, the surgery was performed skillfully, but resulted in blindness in both eyes, a risk about which Client was never informed and a reasonable person would not have known. If Client had known of the risk of blindness, he would not have opted for the surgery. Client hires Attorney Matthew to sue Ophthalmologist for medical malpractice. Attorney Matthew fails to file the complaint to initiate the cause of action against Ophthalmologist within the applicable statute of limitations. As a result, the court dismisses Client's case against Ophthalmologist. Client then hires Attorney Becky to represent him in a cause of action against Attorney Matthew for legal malpractice. In the legal malpractice action brought by Client against Attorney Matthew, what duty applies to Attorney Matthew? A) Attorney Matthew had a duty to act as the average lawyer under the same or similar circumstances B) Attorney Matthew had a duty to act as the reasonable prudent lawyer under the same or similar circumstances C) Attorney Matthew had a duty to act as the reasonable prudent lawyer in the same or similar locality D) Attorney Matthew had a duty to act as the reasonable lawyer with Attorney Matthew's same training and experience

B) Attorney Matthew had a duty to act as the reasonable prudent lawyer under the same or similar circumstances

Plaintiff Olaf, a novice high school hockey player, was struck by an airborne hockey puck and injured. He sued defendant Frozen Fans, Inc., the organizer of the hockey tournament, and alleged that his injuries resulted from its negligence. Which of the following would be Frozen Fans, Inc.'s strongest defense? A) Plaintiff knew of the risk of being struck by a hockey puck and unreasonably and voluntarily encountered it B) Because being struck by a hockey puck is an inherent danger of playing the game of hockey, Frozen Fans owed no duty to Plaintiff C) Plaintiff was contributorily negligent D) The player who fired the shot was the sole proximate cause of Plaintiff's injuries

B) Because being struck by a hockey puck is an inherent danger of playing the game of hockey, Frozen Fans owed no duty to Plaintiff

Company hires Janitor to provide maintenance and janitorial services for its apartment building. When it hires Janitor, Company knows that Janitor has a record of inappropriate sexually aggressive conduct toward women. In his role as janitor, Janitor has frequent interactions with Tenant, one of the building's occupants. One evening, after his workday is over, Janitor knocks on Tenant's door. Being familiar with him because he is the janitor at apartment complex, Tenant lets him in, whereupon Janitor rapes her. Tenant sues Company in negligence, seeking to hold it liable for her injuries. Can Company be held liable for negligence? A) Company may not be held liable in negligence because Janitor's rape of Tenant was an intervening criminal act B) Company may be held liable in negligence in hiring Janitor for a job in which Janitor would have access to female tenants within their apartments C) Company may not be held liable in negligence because Janitor's assault of Tenant was outside the scope of his employment D) Company may be held liable in negligence because employers are liable in negligence for the harm caused by their employees

B) Company may be held liable in negligence in hiring Janitor for a job in which Janitor would have access to female tenants within their apartments

A state administrative regulation prohibits railroad trains from blocking highway crossings for more than 10 minutes. Railway allows one of its trains to remain in a highway crossing for 30 minutes. Fifteen minutes into this 30-minute period, Driver, who is driving his car on the highway, fails to notice the train until it is too late and collides with it, suffering an injury. Driver sues Railway in negligence. At trial, Driver concedes that his negligence should reduce his recovery under comparative negligence principles, but claims that Railway is negligent per se for violating the regulation. The evidence at trial reveals that the history of the regulation and of the agency's findings accompany the regulation shows that the only purpose of the regulation is to encourage the free flow of traffic and prevent traffic delays. What effect, if any, does Railroad's violation of the regulation have on Railroad's liability for negligence? A) Driver should be able to use evidence of the regulation's violation to establish that Railway's negligence because it is a state administrative regulation applicable to railroad trains B) Driver should not be permitted to use the regulation's violation to establish Railway's negligence because the prevention of personal injuries is not part of the regulation's purpose C) Driver may not use the regulation's violation as proof of Railway's negligence per se because Driver has conceded his own negligence D) Driver should not be permitted to use evidence of the regulation's violation to establish that Railway was negligent because administrative regulations may not be used to establish the standard of care in a negligence action

B) Driver should not be permitted to use the regulation's violation to establish Railway's negligence because the prevention of personal injuries is not part of the regulation's purpose

Michael and Jessica, both college-aged environmental activists, entered a coastline city park after midnight to use purple spray paint to paint "End Global Climate Change" on a bluff above the ocean. Later, while walking on a path adjacent to a stone wall marking the edge of the bluff, the ground gave way, and Michael fell to the bottom of the bluff, suffering physical harm. A city ordinance prohibits entry into the park from dusk until dawn, but no physical barrier prevents entry during that time and there are many places where people enter the park that are not marked by a sign stating the hours that the park is open. Michael sues the city for negligence. Under the Restatement (Third) of Torts, which of the following statements about Michael's status and the City's duty to Michael is correct? A) Michael is a trespasser to whom City owes a duty of reasonable care B) Michael is a flagrant trespasser to whom City owes a duty only to avoid harm caused by intentional, willful, or wanton conduct unless he is imperiled, helpless, or unable to protect himself C) Michael is a licensee to whom City owes a duty to warn of risk of which it is aware or has reason to be aware of that are likely not to be discovered by the licensee D) Michael is an invitee to whom City owes a duty to keep the premises reasonably safe

B) Michael is a flagrant trespasser to whom City owes a duty only to avoid harm caused by intentional, willful, or wanton conduct unless he is imperiled, helpless, or unable to protect himself

Michael and Jessica, both college students, went for a midnight stroll in a city park that winds along the coastline on a bluff above the ocean. While walking on a path adjacent to a stone wall marking the edge of the bluff, the ground gave way, and Michael fell to the bottom of the bluff, suffering physical harm. A city ordinance prohibits entry into the park from dusk until dawn, but no physical barrier prevents entry during that time and there are many places where people enter the park that are not marked by a sign stating the hours that the park is open. Michael sues the city for negligence for failing to inspect and maintain the soundness of the ground underlying the foot path. In most jurisdictions, which of the following statements about Michael's status and the duty owed by the City to Michael is correct? A) Michael is a trespasser to whom City owes a duty of reasonable care B) Michael is a trespasser to whom City owes a duty only to avoid harm caused by intentional, willful, or wanton conduct C) Michael is a licensee to whom City owes a duty to warn of risks of which it is aware or has reason to be aware of that are likely not to be discovered by the licensee D) Michael is an invitee to whom City owes a duty to keep the premises reasonably safe

B) Michael is a trespasser to whom City owes a duty only to avoid harm caused by intentional, willful, or wanton conduct

Ellie, who was employed as an asbestos insulation installer for many years, is killed when a defect in a ground-fault interrupter (GFI) in her bathroom fails. A GFI is designed to protect a person from a dangerous or lethal electrical shock hazard by breaking the electrical current if the electrical device comes into significant contact with water. The GFI in Ellie's bathroom failed to break the electrical circuit after an electrical razor fell in the bathtub, resulting in Ellie's electrocution and death. The autopsy reveals that Ellie died from the electrocution. It also shows that she was in the early stages of mesothelioma, which would have killed her eventually. Mesothelioma is a rare and almost invariably fatal form of cancer that studies have established is caused only by exposure to asbestos. In a lawsuit against the manufacturer of the GFI, what is the factual cause of Ellie's death? A) The defect in the GFI is not a factual cause of Ellie's death because she was suffering from a fatal form of cancer at the time and would have died from it eventually B) The defect in the GFI is a factual cause of Ellie's death C) The defect in the GFI and the asbestos in the insulation were both a factual cause of Ellie's death D) Neither the GFI nor the asbestos in the insulation was a factual cause of Ellie's death

B) The defect in the GFI is a factual cause of Ellie's death

Hiker was hiking in the woods. As hiker approached a bend in the path a large tree branch snapped, hitting hiker and sending her airborne for about 30 yards. She landed relatively unhurt but in the process, was thrown over the unmarked property line of landowner, landing on landowner's land damaging some shrubs. If landowner sues for trespass to land, will landowner win or lose? a) Landowner will win because hiker tangibly entered onto her property b) Hiker will win because her entry onto landowner's property was not volitional c) Landowner will lose because the price of shrubs pales in comparison to human life d) Hiker will lose because her entry onto the land caused actual damage

B) hiker will win because her entry onto landowner's property was not volitional

A sues B and C in negligence. B and C defend, claiming that A's negligence contributed to A's injuries. At trial, the jury assigns 40% responsibility to A, 40% responsibility to B, and 20% responsibility to C. It also determines that A's damages are $100,000. In a pure comparative fault jurisdiction, what amount, if any, may A recover? A) A may not recover for her injuries because the jury determined that A was at fault B) A may recover from B and C for a total of $100,000 C) A may recover from B and C for a total of $60,000 D) A may recover from B and C for a total of $40,000

C) A may recover from B and C for a total of $60,000

Abe, who is negligently driving while intoxicated, is stopped at a red light. Betty negligently fails to stop and hits Abe's car in a rear-end collision. Abe sues Betty in negligence for personal injuries he suffered in the collision. In this comparative negligence jurisdiction, what effect will Abe's negligence have on his recovery? A) Abe cannot recover in negligence from Betty because he was intoxicated at the time B) Abe's own negligence of being intoxicated will affect his recovery because his intoxication was a factual cause of his own injuries C) Abe's own negligence of being intoxicated does not affect Abe's recovery because Abe's intoxication was not a factual cause of his own injuries D) Abe's own negligence of being intoxicated will affect his recovery because his intoxication was a proximate cause of his own injuries

C) Abe's own negligence of being intoxicated does not affect Abe's recovery because Abe's intoxication was not a factual cause of his own injuries

Jessica was driving her car, which was manufactured by Motor Company, when the voltage regulator in the car failed due to its negligent installation. The failure caused the battery fluid to boil, which produced toxic fumes that reached the interior of the car. Jessica suffered chronic vocal-cord damage as a result. Prior to this occurrence, Jessica was a popular singer who earned several millions of dollars each year. Jessica sues Motor Company for its negligence to recover for her injuries. Will Jessica be able to recover for her injuries? A) Jessica will only be able to recover for injuries that the ordinary person would have suffered due to the same occurrence, as Jessica being a popular singer was unforeseeable to Motor Company B) Motor Company is only subject to liability for damages to the vehicle itself, not for injuries to a person C) All of Jessica's lost earnings due to her vocal-cord injury are within the scope of Motor Company's liability because the type of injuries Jessica suffered were foreseeable to Motor Company from its negligence D) None of Jessica's lost earnings due to her vocal-cord injury are within the scope of Motor Company's liability because those injuries were not foreseeable to Motor Company from its negligence

C) All of Jessica's lost earnings due to her vocal-cord injury are within the scope of Motor Company's liability because the type of injuries Jessica suffered were foreseeable to Motor Company from its negligence

Austin suffers from Virus-A which causes a disabling disease that although once rare is now relatively common in the United States. He recently noted that the Centers for Disease Control and Prevention (CDC), a federal agency, spends only ten percent as much money researching Virus-A and possible cures for it as it does in researching Virus-B. Apparently this is because the child of the chairman of the congressional Appropriations Committee suffers from Virus-B. Austin has filed an action against the federal government under the Federal Tort Claims Act alleging that the government is negligent in allocating a disproportionately modest amount of funds to research Virus-A. What result is most likely and why? A) Austin will recover because an application of cost-benefit analysis shows that the government is negligent B) Austin will not be able to recover because under the principle of sovereign immunity, the federal government can never be sued in tort C) Austin will not be able to recover because the CDC's decisions as to how to allocate funds for research among various diseases is a discretionary function D) Austin will not be able to recover because the actions of the committee chair are protected by the Speech and Debate Clause of the U.S. Constitution

C) Austin will not be able to recover because the CDC's decisions as to how to allocate funds for research among various diseases is a discretionary function

In a negligence action brought by Plaintiff Betty against Defendant Alice and Defendant Carl, the jury apportions liability based on the negligence of the parties as follows: Betty is 20% at fault; Alice is 60% at fault; and Carl is 20% at fault. The jury also determines that Plaintiff Betty sustained $100,000 in compensatory damages. The parties are in a modified "not as great as" comparative fault jurisdiction that provides that defendants are jointly and severally liable and the plaintiff's fault is compared to each defendant separately. What amount may Betty recover from Carl or Alice? A) Betty may recover 100% of $80,000 from Carl or Alice or a combination of both B) Betty may recover 100% of $100,000 from Carl or Alice or a combination of both C) Betty may not recover from Carl, but can recover 100% of $80,000 from Alice D) Betty may not recover from Carl, but can recover 100% of $100,000 from Alice

C) Betty may not recover from Carl, but can recover 100% of $80,000 from Alice

Connie is carelessly changing lanes in her car when her car is struck by Bill who is reaching for his phone, which has fallen under the passenger side seat while he was driving. Adrian, who is in the vicinity on her motorcycle waiting patiently at the intersection for the light to turn green, is struck by Connie's car and suffers serious physical injury as well as damage to her bike. She sues both Connie and Bill for their negligence. The jury determines that Adrian's damages are $100,000 and concludes that Bill and Connie are both liable. It assigns 30% responsibility to Bill, and 70% responsibility to Connie. In a jurisdiction that subjects defendants to joint and several liability, if Adrian recovers $100,000 from Bill alone, what amount is Bill entitled to collect from Connie? A) Bill is entitled to collect $100,000 in contribution from Connie B) Bill is entitled to collect $100,000 in satisfaction from Connie C) Bill is entitled to collect $70,000 in contribution from Connie D) Bill is entitled to collect $70,000 in satisfaction from Connie

C) Bill is entitled to collect $70,000 in contribution from Connie

Corky Codger knows that each morning and each afternoon during the school year, middle-school students cut across a corner of his suburban yard on their way to and from school. Although he has never told the children explicitly that he approves of their use of his yard for a shortcut, he quietly tolerates it. One September morning, one of the students, Briar Rose, steps into a nest of yellow jackets underneath the lawn in Codger's yard. Upset by the intruder, numerous yellow jackets sting Briar Rose, injuring her. Codger knew nothing about the nest. He avoids going outside during the summer because of his allergies, and he hires a lawn maintenance contractor to take care of his lawn. If Codger had inspected his lawn regularly, he would have discovered the nest and had it removed. In a jurisdiction that follows the traditional rules for classifying land visitors as invitees, licensees, or trespassers, into which category would Briar Rose fit and what duty of care would Codger owe her? A) Briar Rose is a trespasser because Codger had not consented to her presence on his land, and thus Codger would only owe her a duty to refrain from intentional, willful, or wanton conduct causing harm to her B) Even though Briar Rose is a trespasser, because she is a child, she is owed a duty of reasonable care C) Briar Rose is a licensee or a discovered trespasser because Codger tolerated her presence on his land. Codger owed Briar Rose only a duty to warn her of dangers which he knew about or had reason to know about that were unlikely to be discovered by Briar Rose D) Briar Rose is a public invitee because she is using Codger's property for a purpose for which it is being held open to the public

C) Briar Rose is a licensee or a discovered trespasser because Codger tolerated her presence on his land. Codger owed Briar Rose only a duty to warn her of dangers which he knew about or had reason to know about that were unlikely to be discovered by Briar Rose

Defendant, driving a car, approaches Plaintiff's car, and properly steps on the brakes. Defendant's brakes fail to function because she carelessly maintained the brakes. Defendant's car continues forward. In this emergency, Defendant chooses to step on the brakes again, and they fail again, causing Defendant's car to strike Plaintiff's car, injuring Plaintiff. Will Defendant likely be held liable for negligence? A) Defendant will likely not be held liable for negligence because in light of her emergency circumstances she behaved as a reasonable prudent person B) Defendant will likely be held liable in negligence because she should have chosen better by turning the car quickly to the right C) Defendant will likely be held liable in negligence because her brakes failed because she unreasonably failed to properly maintain her brakes D) Defendant will likely not be held liable in negligence because she was reacting to an emergency situation

C) Defendant will likely be held liable in negligence because her brakes failed because she unreasonably failed to properly maintain her brakes

In a lawsuit brought by Plaintiff against Defendant alleging negligence, Plaintiff seeks to use Defendant's violation of a statute to help establish his prima facie case. The court conducts an inquiry to determine: (1) whether Plaintiff is a member of the class the legislature intended to protect in enacting the statute; (2) whether the hazard about which Plaintiff is complaining is one the legislature intended to prevent; and (3) whether tort liability is appropriate. How is conducting this three step analysis important to Plaintiff's claim against Defendant? A) It establishes whether Plaintiff may use the doctrine of res ipsa loquitur B) It establishes the effect of proof of the statute's violation C) It establishes whether the statute's violation can be used to establish negligence (also known as negligence per se) D) It establishes whether a Rule of Law may be used to establish negligence

C) It establishes whether the statute's violation can be used to establish negligence (also known as negligence per se)

While visiting her friend Tyler Tenant, Jean Guest asked to use the bathroom. Tyler told her that it was upstairs and directed her to the stairwell. Earlier in the week Tyler had told Lauren Landlord, the owner of the apartment building, that the railing in the stairwell was loose and needed to be repaired. Lauren told Tyler that she would have it fixed but that her maintenance person was already booked so that it would take a couple of weeks. In the midst of his pleasant evening hosting Jean, Tyler forgot all about the railing and neglected to tell her about it. Jean was injured when the railing gave way. She sues both Tyler Tenant and Lauren Landlord for negligence. From whom will she be able to recover and why? A) Jean will be able to recover only from Tyler Tenant. He was negligent because he was aware of the loose railing and failed to warn her. Lauren Landlord is not liable because the general rule is that the landlord is not liable for injuries caused by conditions in leased premises, sometimes known as the "landlord-out-of-possession" doctrine B) Jean will be able to recover only from Lauren Landlord, but not from Tyler Tenant. Tyler acted reasonably by informing Lauren of the loose railing and once he had done this, it was Lauren's responsibility, not his, to have the railing repaired C) Jean will recover from both Tyler Tenant and Lauren Landlord. Under the traditional classification system, Tyler, as the possessor of land, owes a duty to his social guest to warn her of any dangers that he is aware of or has reason to be aware of that are unlikely to be discovered by a social visitor. Today, in approximately half of all jurisdictions, he owes a duty of reasonable care to his visitors. Under either standard, Tyler breached a duty of care by failing to warn Jean of the loose railing. Lauren Landlord also is liable. Although some jurisdictions still follow the "landlord-out-of-possession doctrine," there is an exception when the landlord has agreed to repair something that subsequently causes harm to a visitor D) Jean will recover from neither Tyler Tenant nor from Lauren Landlord. As a social guest, she qualifies neither as a business visitor nor as a public invitee, and therefore Tyler and Lauren only owe her only a duty to avoid harm caused by intentional, willful, or wanton conduct

C) Jean will recover from both Tyler Tenant and Lauren Landlord. Under the traditional classification system, Tyler, as the possessor of land, owes a duty to his social guest to warn her of any dangers that he is aware of or has reason to be aware of that are unlikely to be discovered by a social visitor. Today, in approximately half of all jurisdictions, he owes a duty of reasonable care to his visitors. Under either standard, Tyler breached a duty of care by failing to warn Jean of the loose railing. Lauren Landlord also is liable. Although some jurisdictions still follow the "landlord-out-of-possession doctrine," there is an exception when the landlord has agreed to repair something that subsequently causes harm to a visitor

Jeff is driving 35 miles per hour on a city street with a speed limit of 25 miles per hour with Nate as his passenger. But for his speeding, Jeff would not have been at the very place on the road when, without warning, a tree crashes onto Jeff's car, injuring Nate. In a negligence action brought by Nate against Jeff, should Jeff be liable for negligence? A) Jeff should not be liable in negligence because Jeff's speeding is not a factual cause of Nate's harm B) Jeff should be liable in negligence because Jeff's negligent conduct of speeding increased the risk of the type of harm suffered by Nate C) Jeff should not be liable in negligence because Jeff's negligent conduct of speeding did not increase the risk of the type of harm suffered by Nate D) Jeff should not be liable in negligence because a passenger of a car cannot successfully sue the driver of the car in negligence

C) Jeff should not be liable in negligence because Jeff's negligent conduct of speeding did not increase the risk of the type of harm suffered by Nate

Brenda Botulism is a state health inspector. Recently, while driving to inspect a restaurant as part of her duties, she skidded on wet pavement and ran into a car driven by Logan Luckless, injuring him. At the time of the accident, Brenda was driving significantly in excess of the speed limit despite the wet road conditions. Logan sues Brenda and her employer (the state), alleging negligence under the state tort claims act that contains the provisions typically present in such acts. Which result is most likely and why? A) Logan will lose in his claims against both Brenda and the state because Brenda, a regulatory official, was engaged in a discretionary act B) Logan will prevail in his claim against Brenda personally, but lose in his action against the state because Brenda had been told never to drive in excess of the speed limit while on state business C) Logan will lose in his claim against Brenda because most state tort claims act grant state employees personal immunity from liability for torts committed while conducting state business unless the tort is particularly egregious. However, Logan will be able to recover from the state because the state tort claims act makes the state vicariously liable for the negligent acts of its employee when engaged in a ministerial function D) Logan will recover against both Brenda and the state

C) Logan will lose in his claim against Brenda because most state tort claims act grant state employees personal immunity from liability for torts committed while conducting state business unless the tort is particularly egregious. However, Logan will be able to recover from the state because the state tort claims act makes the state vicariously liable for the negligent acts of its employee when engaged in a ministerial function

Mom is the mother of Toddler, who is almost two years old. Mom and Toddler are visiting Friend's quaint rustic cabin. One morning when Friend has gone to run errands, Mom and Toddler are in the kitchen, a lovely room lit by a kerosene lamp sitting on a kitchen table. Toddler knocks the lantern over during a less than 10-second period during which Mom has turned her back in order to take a boiling pot off the stove. The knocked over lantern starts a fire that damages Friend's cabin. In a negligence action brought by Friend against Mom, is Mom likely to be held liable in negligence? A) Mom is likely not to be held liable in negligence because Toddler caused the fire, not her B) Mom is likely liable for negligence if she failed to act as the reasonable prudent person when she left Toddler alone next to a kerosene lamp. C) Mom is likely not liable in negligence if turning her back for that brief period of time was reasonable. D) Mom is likely liable in negligence because parents are vicariously responsible for the damage caused by their children

C) Mom is likely not liable in negligence if turning her back for that brief period of time was reasonable

Drug Manufacturers (a total of seven companies) manufactured, produced, and marketed a medication, DRG, designed to prevent miscarriages in pregnancy. The drug was administered to pregnant women. Unfortunately, women who ingested DRG, thereby exposing their unborn daughters to DRG, gave birth to daughters who developed cancerous vaginal and cervical growths. The unique type of cancer these daughters developed was a fast-spreading and deadly disease requiring radical surgery to prevent its spread. Daughter's mother ingested DRG, exposing Daughter to the drug prior to her birth. The evidence also shows that this unique type of cancer is most probably caused by exposure to DRG. Daughter has developed the cancerous vaginal and cervical growths, which are only developed by women like her whose moms ingested DRG. Daughter has sued Drug Manufacturers, all of whom manufactured, produced, and marketed DRG at the time that Daughter's mother ingested the drug. Drug Manufacturers represent 90% of the market share of manufacturers of DRG. Daughter seeks to hold them responsible in negligence for their manufacture of DRG and Daughter's resultant injuries. However, Daughter is unable to prove with certainty which one of the seven defendants actually manufactured the DRG that her mother ingested. Drug Manufacturers have moved for summary judgment, arguing that Daughter's cause of action against them should be dismissed for failing to establish factual causation. In a jurisdiction that recognizes market share liability, should the court grant defendants' motion for summary judgment? A) Yes, evidence that Drug Manufacturers produced the drug to which Daughter was exposed is insufficient to establish factual case B) No, Drug Manufacturers produced the drug at the time that Plaintiff was exposed to DRG in utero and therefore are responsible for her injuries C) No, as long as Defendants represent a substantial share of the market of those manufacturing DRG and at the time Daughter's mother ingested DRG, each defendant may be responsible D) Yes, defendants cannot be subject to liability for injuries caused by a drug to which Plaintiff was exposed prior to her birth

C) No, as long as Defendants represent a substantial share of the market of those manufacturing DRG and at the time Daughter's mother ingested DRG, each defendant may be responsible

Owner owns an apartment building and employs Manager as its on-site manager. Owner reasonably believes that Manager's temperament is suited to the duties of on-site apartment management based on Owner's inquiries to Manager's prior employers. Owner does not check with law enforcement authorities. Had Owner done so, Owner would have discovered that Manager has been convicted of income-tax evasion, but Owner would have hired him in any event because Owner would not have viewed that conviction as material to Manager's duties as an on-site manager, a job which does not involve tax-compliance tasks. One day, Tenant complains to Manager about the lack of heat in Tenet's apartment. When Manager visits Tenant about the complaint, Manager becomes enraged and assaults Tenant. In a lawsuit brought by Tenant against Owner for his negligence, may owner be subject to liability for Manager's assault of Tenant A) Owner may be subject to liability for Manager's assault of Tenant because Manager was Owner's employee B) Owner may not be subject to liability to Tenant as a consequence of Manger's assault on Tenant because an employer cannot be subject to liability for the tortious conduct of its employees C) Owner may not be subject to liability to Tenant as a consequence of Manager's assault on Tenant because Owner's failure to detect Manager's tax-evasion conviction was not a factual cause of Tenant's harm and did not increase the risk of an assault by Manager against Tenant D) Owner may be subject to liability for Manger's assault of Tenant because Owner was negligent in failing to discover accurate information about Manager's temperament

C) Owner may not be subject to liability to Tenant as a consequence of Manager's assault on Tenant because Owner's failure to detect Manager's tax-evasion conviction was not a factual cause of Tenant's harm and did not increase the risk of an assault by Manager against Tenant

A state statute requires that all slow-moving vehicles drive as far to the right on the highway as possible. The statute's primary purpose is to minimize the safety hazards posed by vehicles whose slow speed can interfere with the traffic flow. Driver is driving slowly on a highway heading north and is violating the statute by driving in the fast lane on the left. Passenger is riding along in her car. Unexpectedly, a car heading south on the highway crosses the median line striking Driver's car and injuring Passenger. Had Driver been in the lane on the far right, her car would have avoided contact with the other car. Passenger sues Driver, seeking to establish her negligence by proving her violation of the statute. What effect, if any, may proof of Driver's violation of the statute have in a negligence action brought by Passenger? A) Driver's violation of the statute is negligence per se because the purpose of the statute is to protect highway safety B) Driver's violation of the statute may be used to establish the standard of care because any statute may be used to establish the standard of care in a negligence action C) Passenger will likely not be able to use the statute to prove Driver's negligence because the statute was designed to prevent accidents between cars moving in the same direction, not to prevent accidents between cars moving in opposite directions D) Passenger will likely be able to use the statute to establish the standard of care in negligence because if she had not been violating the statute, Passenger would not have been injured

C) Passenger will likely not be able to use the statute to prove Driver's negligence because the statute was designed to prevent accidents between cars moving in the same direction, not to prevent accidents between cars moving in opposite directions

Paula Passenger, riding in a car driven by David Driver, was seriously injured in an accident resulting from the negligence of defendant Driver. Plaintiff Passenger was not wearing a seat belt. As a result, plaintiff sustained severe injuries. If plaintiff had been wearing her seat belt, her injuries would have been minor. Plaintiff sues defendant Driver in a comparative fault jurisdiction. What result? A) Plaintiff will be able to recover for all her damages because any contributory negligence or other fault on her part was not a factual or proximate cause of the accident B) Plaintiff will be able to recover damages only for her minor injuries and not for the more severe injuries resulting from her failure to wear a seat belt C) Plaintiff's failure to wear her seat belt will be factored into a comparative fault analysis and she will recover the portion of all the damages suffered that is attributable to defendant's degree of fault D) Plaintiff cannot recover damages because of her contributory negligence

C) Plaintiff's failure to wear her seat belt will be factored into a comparative fault analysis and she will recover the portion of all the damages suffered that is attributable to defendant's degree of fault

Officer Odo visited the home of Suspect Sith to execute a search warrant. As he stood knocking on the front door, the wooden porch beneath his feet gave way and Officer Odo was injured. As his police colleagues investigated the scene, it became clear that the boards in the porch had rotted and a reasonable inspection would have revealed the structural weakness of the porch. Odo sues Sith for negligence in any of the approximately one-half of all jurisdictions that follow the Restatement (Third) of Torts. What standard will the court apply and what outcome is most likely? A) The court will find that Odo is an invitee and that Sith owed a duty of reasonable care including a duty to reasonably inspect the premises for hazardous conditions B) The court will find that Odo is a licensee because police officers and firefighters are usually regarded as licensees. Sith owed a duty to warn of any dangers of which he was aware or had reason to know about but that were not likely to be discovered by a visitor C) The court will apply a general negligence standard and find that Sith failed to use reasonable care to prevent injury to Odo and other visitors D) The court will apply the "firefighter's rule" and hold that Sith cannot recover

C) The court will apply a general negligence standard and find that Sith failed to use reasonable care to prevent injury to Odo and other visitors

Farmer is a country farmer who has a lower IQ than most individuals. However, he is also unusually physically fit, much physically bigger and stronger than the average person. Lately, as part of his trade as a farmer, he has been stacking hay on his property. However, he has been doing so in a way that a farmer of ordinary intelligence would know will cause the hay to combust spontaneously into flames. Defendant's hay does in fact combust causing a fire, which spreads to Neighbor's property causing property damage. In the negligence action brought by Neighbor against Farmer, how should the court instruct the jury regarding the reasonable prudent person standard of care? A) The reasonable prudent person standard will take into account that Farmer has a lower IQ than most individuals B) The reasonable prudent person standard will not take into account that Farmer was physically bigger and stronger than the average person C) The reasonable prudent person standard will not take into account that Farmer has a lower IQ than most individuals D) The reasonable prudent person standard is subjective and based on Farmer's qualities

C) The reasonable prudent person standard will not take into account that Farmer has a lower IQ than most individuals

About two years ago, Client was a patient of Ophthalmologist. Client alleges that during the time that he was Ophthalmologist's patient, Ophthalmologist improperly performed LASIK surgery on Client's eyes. The LASIK surgery was supposed to correct Client's poor vision. Instead, the surgery was performed skillfully, but resulted in blindness in both eyes, a risk about which Client was never informed and a reasonable person would not have known. If Client had known of the risk of blindness, he would not have opted for the surgery. Client hires Attorney Matthew to sue Ophthalmologist for medical malpractice. Attorney Matthew fails to file the complaint to initiate the cause of action against Ophthalmologist within the applicable statute of limitations. As a result, the court dismisses Client's case against Ophthalmologist. Client then hires Attorney Becky to represent him in a cause of action against Attorney Matthew for legal malpractice. In the legal malpractice action brought by Client against Attorney Matthew, how, if at all, must Client prove factual causation? A) Client will not need to prove factual causation in this legal malpractice action B) To prove factual causation in the legal malpractice action, Client will need to prove that he possibly would have prevailed in the underlying medical malpractice action C) To prove factual causation in the legal malpractice action, Client will need to prove that he more likely than not would have prevailed in the underlying medical malpractice action D) To prove factual causation in the legal malpractice action, Client will need to prove that Attorney Matthew breached the standard of care in not filing the complaint before the statute of limitations ran

C) To prove factual causation in the legal malpractice action, Client will need to prove that he more likely than not would have prevailed in the underlying medical malpractice action

Toddler, a two-year-old child suffering from a bacterial infection, received a routine childhood vaccination with a vaccine manufactured by Vaco. Shortly after receiving the vaccine, Toddler spiked a very high fever, went into respiratory arrest, and died. Toddler's estate sues Vaco in negligence. At trial, Toddler's estate provides competent evidence that Toddler died due to a combination of the infection and the vaccine, and if either toddler had not had an infection or if Toddler had not been vaccinated, death would not have ensued. In its defense, Vaco presents evidence that the infection and its drug do not react together, but rather are entirely independent. Therefore, Toddler's death could only have been caused by the infection or the drug, but not a combination of both. The factfinder is persuaded by the defense's evidence and finds that the infection more likely than not caused the death of Toddler rather than the vaccine. Which of the following statements regarding the factual causation element of the negligence action is correct? A) Toddler's estate has not provided evidence on the issue of factual causation B) Toddler's estate has satisfied its burden of persuasion that the vaccine was a factual cause of Toddler's death C) Toddler's estate has failed to satisfy its burden of persuasion that the vaccine was a factual cause of Toddler's death D) Vaco may be held liable for the death of Toddler even without proof of factual causation

C) Toddler's estate has failed to satisfy its burden of persuasion that the vaccine was a factual cause of Toddler's death

Plaintiff is injured when Defendant briefly loses control of the car he was driving when he reaches for his cell phone he dropped on the floorboard and hits Plaintiff who was standing on the street corner. Witness was a passenger in the Defendant's car at the time of the accident and saw everything that happened. In the negligence action brought by Plaintiff against Defendant to recover for her injuries, Witness testifies that she was riding in Defendant's car while Defendant was driving, that she heard Defendant's cell ring, that Defendant went to answer his cell phone while he was driving but dropped it on the floorboard of the car, and that when Defendant went to pick up his cell phone while he was driving, he took his eyes off the road for a brief period of time during which he lost control of the car and struck Plaintiff. What is the proper characterization of the evidence presented at Plaintiff's trial against Defendant for Defendant's alleged negligence? A) Witness's testimony is circumstantial evidence of Defendant's conduct that may be used to prove that Defendant breached the standard of care B) Witness's testimony is circumstantial evidence of Defendant's conduct that may not be used to prove that Defendant breached the standard of care C) Witness's testimony is direct evidence of Defendant's negligence and does not require the factfinder to draw an inference regarding Defendant's breach of the standard of care D) Witness's testimony is direct evidence of Defendant's negligence and requires the factfinder to draw an inference regarding the Defendant's breach of the standard of care

C) Witness's testimony is direct evidence of Defendant's negligence and does not require the factfinder to draw an inference regarding Defendant's breach of the standard of care

Visitor was a customer of Barge, when while on the deck of Barge he accidentally fell into the water as the barge approached the pier. Employee of the barge hears Visitor's cries as he yells from the water that he cannot swim. Employee threw a heavy line (used to tie the barge to the pier) in the direction of the cries. The line came within two feet of the spot of Visitor, but Visitor did not grab the lines. Visitor was carried away from the pier by the outgoing tide and downed. At the time of this incident, the employee was aware that several life preservers were on a rack located near the front of the barge where Visitor fell overboard. The preservers remained in the rack and were not used. If the preservers had been used, Victim probably would not have drowned. Visitor's estate sues Barge in negligence, alleging that the failure to use the life preservers was the factual cause of Victim's drowning. Is Visitor's estate correct that Barge's failure to use the life preservers was a factual cause of Victim's drowning? A) No because the failure to use the life preservers is an omission and an omission cannot be the basis for a negligence action B) No because it is possible that Victim would have drowned even if the life preservers had been thrown out to him C) Yes because not throwing the preservers to Victim was a substantial factor in bringing about Victim drowning D) Yes because Victim drowned when he was a customer of Barge

C) Yes because not throwing the preservers to Victim was a substantial factor in bringing about Victim drowning

Curtis was a guest at Hotel where at 9:00 p.m. one evening there was a power outage that deprived Hotel of electricity. As there was no emergency lighting in his dark room, Curtis fell and injured himself. Curtis sued Hotel for his injuries, alleging that Hotel was negligent in not having inexpensive battery-powered lighting fixtures installed in its guest rooms. Curtis alleges that if his room had such a fixture, he would have avoided injury. The only expert to testify at trial explains that at the time of Curtis's injury, it was not customary in the hotel industry for in-room emergency lighting of this sort to be provided. May Hotel be subject to liability in negligence although if it was in compliance with customary hotel industry standards by not providing in-room emergency lighting? A) No, the evidence of Hotel's compliance with customary hotel industry standards, by not providing in-room emergency lighting, is conclusive proof that Hotel was not negligent B) Yes, the evidence of Hotel's compliance with customary hotel industry standards is legally irrelevant in determining whether Hotel was negligent C) Yes, Hotel may be liable for Curtis's injuries despite the industry custom if requiring emergency lighting in hotel rooms is reasonable with respect to the risk of guests injuring themselves during a power outage D) No, Hotel may be liable for Curtis's injuries despite the industry customs because Curtis was injured in one of Hotel's room through no fault of his own

C) Yes, Hotel may be liable for Curtis's injuries despite the industry custom if requiring emergency lighting in hotel rooms is reasonable with respect to the risk of guests injuring themselves during a power outage

As Plaintiff is walking down the street, a chair falls out of Warehouse's window, striking Plaintiff on the head. Plaintiff seeks to sue Warehouse in negligence and wants to employ the doctrine of res ipsa loquitur to do so. When using the doctrine of res ipsa loquitur to prove a Warehouse's negligence, Plaintiff: A) most likely has additional direct evidence of the defendant's negligence B) most likely has additional circumstantial evidence of the defendant's negligence C) must prove that the accident does not ordinarily occur in the absence of negligence D) must specifically identify what the defendant's negligent act entailed

C) must prove that the accident does not ordinarily occur in the absence of negligence

Two college students, A & B, were out drinking heavily one night. A third college student, C, who knew the drunk students, watched as A and B passed out. C drove B to B's house and left B there. After doing this C thought it would be a great idea to drop the unconscious A on some random stranger's lawn. C pulled up next to an unfenced yard and dragged A onto the lawn. The next morning the irate property owner woke up and asked A to leave. Did A and/or C trespass? a) Neither A nor C trespassed b) Only A trespassed c) Only C trespassed d) Both A and C trespassed

C) only C trespassed

Alice sues Bill alleging negligence. Bill defends, claiming that Alice's negligence also contributed to her injuries. At trial, the factfinder assigns 50% responsibility to both Alice and Bill. The factfinder also finds that Alice's damages are $100,000. In a modified comparative fault (50% or "not as great as") jurisdiction, what amount, if any, is Alice entitle to recover from Bill? A) Alice is entitled to recover $50,000 from Bill because Bill only has to pay for his percentage of her damages attributable to his fault B) Alice is entitled to recover $50,000 from Bill because her recoverable damages are the judgment reduced by the amount of fault attributable to her C) Alice is entitled to recover $100,000 from Bill D) Alice is completely barred from recovering from Bill because she was 50% at fault

D) Alice is completely barred from recovering from Bill because she was 50% at fault

Alice sues Bill alleging negligence. Bill defends, claiming that Alice's negligence also contributed to her injuries. At trial, the factfinder assigns 50% responsibility to both Alice and Bill. The factfinder also finds that Alice's damages are $100,000. In a traditional contributory negligence jurisdiction, what amount, if any, is Alice entitled to recover? A) Alice is entitled to recover $50,000 from Bill because Bill only has to pay for his percentage of the judgment attributable to his fault B) Alice is entitled to recover $50,000 from Bill because her recoverable damages are her damages reduced by the amount of fault attributable to her C) Alice is entitled to recover $100,000 from Bill D) Alice is completely barred from recovering from Bill because she was partially at fault

D) Alice is completely barred from recovering from Bill because she was partially at fault

Alice, Bill, and Charles, each acting independently but simultaneously, negligently lean on Polly's car, which is parked at an overlook at the edge of a mountain. Their combined force results in Polly's car rolling over the edge of the cliff and plummeting down the mountain to its destruction. The force exerted by either Alice, Bill, or Charles would have been insufficient alone to propel Polly's car over the edge, but the combined force of any two of them would have been sufficient. In a lawsuit brought by Polly against Alice, Bill, and Charles based on negligence, who is the factual cause of the destruction of Polly's car? A) Neither Alice, Bill, nor Charles is a factual cause of the destruction of Polly's car because the force exerted by each of them alone would have been insufficient to move Polly's car down the mountain B) Neither Alice, Bill, nor Charles is a factual cause of the destruction of Polly's car because the combined force of any two of them would have been sufficient C) Two and only two of Alice, Bill, and Charles are a factual cause of the destruction of Polly's car because the combined force of any two of them would have been sufficient D) Alice, Bill, and Charles are each a factual cause of the destruction of Polly's car because their combined force resulted in the destruction Polly's car

D) Alice, Bill, and Charles are each a factual cause of the destruction of Polly's car because their combined force resulted in the destruction Polly's car

In a negligence action brought by Plaintiff Betty against Defendant Alice and Defendant Carl, the jury apportions liability based on the negligence of the parties as follows: Betty is 20% at fault; Alice is 60% at fault; and Carl is 20% at fault. The jury also determines that Plaintiff Betty sustained $100,000 in compensatory damages. The parties are in a contributory negligence jurisdiction that provides that defendants are jointly and severally liable to plaintiffs. What amount, if any, may Betty recover? A) Betty may recover up to 60% of $100,000 from Alice and up to 20% of $100,000 from Carl B) Betty may recover up to 60% of $80,000 from Alice and up to 20% of $80,000 from Carl C) Betty may recover 100% of $100,000 from either Alice or Carl or a combination of both D) Betty may not recover at all

D) Betty may not recover at all

Chaz is a highly compensated executive of Small Corp. While driving on business for Small Corp., Chaz negligently collides with a car driven by Terri Teacher and injures her. From whom can Teacher recover? A) Chaz alone because he was negligent and he was not at his place of employment at the time of the accident B) Chaz alone because as a highly compensated employee, he was acting as an independent contractor C) Small Corp. alone because Chaz's negligent driving was in the scope of his employment, and therefore Small Corp. is vicariously liable D) Both Chaz and Small Corp. Chaz was negligent, and Small Corp. is vicariously liable for the harm caused by his negligent conduct within the scope of employment

D) Both Chaz and Small Corp. Chaz was negligent, and Small Corp. is vicariously liable for the harm caused by his negligent conduct within the scope of employment

Carl negligently injures himself in an automobile accident. Carl seeks medical treatment from Doctor, who negligently aggravates Carl's injury. In a suit in which Carl seeks to recover from Doctor for the part of Carl's injuries caused by Doctor's medical malpractice, will Carl be able to recover from Doctor? A) Carl cannot recover from Doctor because Carl's negligence caused the automobile accident in which he was initially injured B) Carl may hold Doctor liable for all of his injuries, including the injuries he sustained in the accident Carl caused C) Carl may hold Doctor liable only if Doctor intended to cause Carl harm in providing him with medical care D) Carl may recover for Doctor's negligence for the part of Carl's injury attributable to Doctor's negligence notwithstanding the fact that Carl's negligence produced the very condition Doctor undertook to treat

D) Carl may recover for Doctor's negligence for the part of Carl's injury attributable to Doctor's negligence notwithstanding the fact that Carl's negligence produced the very condition Doctor undertook to treat

Lilly was a guest at Motel, which was located in a neighborhood where significant violent crime existed. After Lilly returned to her hotel room, Don was able to gain entrance to Lilly's room because the lock on the door was of the simple residential type that could be easily defeated with a credit card. After gaining entrance to Lilly's room, Don sexually assaulted her. Lilly sues Motel, claiming that it was negligent in providing inadequate locks for its guest rooms. May Hotel be held liable in negligence? A) Motel may not be held liable in negligence because Don's criminal act was intentional and therefore breaks the chain of causation B) Motel may not be held liable because Don's criminal act was an intervening cause C) Motel may be held liable because Don's act was unforeseeable, unusual, and highly culpable D) Motel may be held liable in negligence because Don's criminal act was foreseeable from its negligence

D) Motel may be held liable in negligence because Don's criminal act was foreseeable from its negligence

Dr. Megan Daniels (MD) is a licensed physician specializing in emergency room medicine. She is also the mother of a rambunctious four-year old daughter, Sofia. On her day off, Dr. Daniels accompanied her daughter to a playground in a nearby park. Sofia slipped off the top of a slide and fell. Dr. Daniels tried to treat Sofia, but because Dr. Daniels had consumed several glasses of wine, she committed medical malpractice and exacerbated Sofia's injuries. Tragically, Sofia will be disabled the remainder of her life, unable to earn a decent income and constantly in need of medical and rehabilitative care. Brent, Sofia's father and Megan's husband, sues Dr. Megan Daniels on Sofia's behalf for negligence in her treatment of Sofia. What result is likely in a majority of jurisdictions and why? A) Dr. Daniels is immune from liability because she is Sophia's parent B) Dr. Daniels is immune from liability under the doctrine of interspousal immunity because the legal action was actually filed by her husband C) Dr. Daniels will be liable because her actions in drinking two glasses of wine and then trying to render medical care are so outrageous as to take her outside the boundaries of parental immunity D) Dr. Daniels will be liable under the dual capacity doctrine because the court will view her relationship with Sofia at the time of the medical treatment as one between a doctor and a patient, not mother and daughter

D) Dr. Daniels will be liable under the dual capacity doctrine because the court will view her relationship with Sofia at the time of the medical treatment as one between a doctor and a patient, not mother and daughter

Danielle Drinker consumed six shots of hard liquor and ten beers on her twenty-first birthday. She then drove the wrong way on a one-way street and injured Robert Responsible. In addition to recovering compensatory damages, Responsible likely will also be entitled to punitive damages because: A) Drinker violated the jurisdiction's driving-under-the-influence statute B) Drinker will be held to an adult standard of care because she was participating in an adult activity C) Drinker will be held to an adult standard of care because she just turned 21 years of age D) Drinker engaged in willful and wanton misconduct

D) Drinker engaged in willful and wanton misconduct

Lincoln's airplane was seriously damaged when he was forced to land it without its retractable gear in the down position. Lincoln sues Dow Aircraft, the manufacturer of the plane. Lincoln claims that Dow Aircraft neglected to include instructions in its service manual of the need to be sure of a minimum clearance between two parts in the landing gear assembly when reassembling the gear after routine servicing. Lincoln's plane was serviced at Dow because Dow needed to complete unrelated warranty work to the aircraft. The Dow mechanic who worked on Lincoln's plane was fired shortly after the work was completed for repeatedly failing to consult service manuals. In a cause of action against Dow Aircraft and the mechanic, Lincoln includes a claim based on the mechanic's negligence. Which of the following statements regarding factual cause of the harm to Lincoln's plane is correct? A) The omitted instruction by itself was a factual cause of the harm to Lincoln's plane B) The mechanic's negligence in failing to consult the manual was by itself a factual cause of the harm to Lincoln's plane C) Because neither the missing instructions nor the mechanic's failure to consult the manual were a factual cause of Lincoln's harm, Lincoln will not be able to establish that Dow Aircraft was a factual cause of the damage to his plane D) If Lincoln can prove both that Dow Aircraft providing the omitted instructions and a mechanic properly consulting the service manual would have prevented the damage to Lincoln's plane, Dow Aircraft's multiple acts of negligence are a factual cause of the plane's damage

D) If Lincoln can prove both that Dow Aircraft providing the omitted instructions and a mechanic properly consulting the service manual would have prevented the damage to Lincoln's plane, Dow Aircraft's multiple acts of negligence are a factual cause of the plane's damage

Parker parks his car at the top of a driveway, which is on an incline. Two minutes later, the car rolls down the incline and injures Pedestrian who was on the sidewalk. In suing Parker, Pedestrian seeks to rely on res ipsa loquitur in order to prove Parker's negligence. Which of the following statements about res ipsa loquitur is correct? A) Relying on res ipsa loquitur in these circumstances would be improper if there are other possible explanations for why the car rolled B) If able to employ the doctrine of res ipsa loquitur, the jury is required to infer that Parker was negligent C) If permitted to employ the doctrine of res ipsa loquitur, the jury may not infer that Parker was negligent D) If permitted to employ the doctrine of res ipsa loquitur, the jury may infer that Parker was negligent

D) If permitted to employ the doctrine of res ipsa loquitur, the jury may infer that Parker was negligent

Jill loans her car for the evening to Friend who needs the car for social purposes. Jill knows that Friend's driver's license was suspended a month previously on account of repeated instances of reckless driving. In the course of the evening, Friend drives the car negligently and injures Pedestrian. In a cause of action against Jill brought by Pedestrian for recovery of her injuries, can Jill be held liable for negligence? A) Jill cannot be held liable in negligence because Friend was driving the car and he was therefore the sole proximate cause for Pedestrian's injuries B) Jill cannot be held liable in negligence because an adult cannot be held liable for the tortious conduct of another adult C) Jill may be held liable to Pedestrian in negligence because car owners are responsible for injuries caused by their vehicles D) Jill may be held liable to Pedestrian in negligence because the reasonable prudent person would likely not have lent her car to Friend, knowing that Friend's license was suspended for repeated instances of reckless driving

D) Jill may be held liable to Pedestrian in negligence because the reasonable prudent person would likely not have lent her car to Friend, knowing that Friend's license was suspended for repeated instances of reckless driving

Mom is the mother of Toddler, who is almost two years old. Mom and Toddler are visiting Friend's quaint rustic cabin. One morning when Friend has gone to run errands, Mom and Toddler are in the kitchen, a lovely room lit by a kerosene lamp sitting on a kitchen table. When Mom leaves the kitchen for an hour to read a book, Toddler knocks over the lantern, starting a fire that damages Friend's cabin. In a negligence action brought by Friend against Mom, is Mom likely to be held liable in negligence? A) Mom is not likely to be held liable to Friend in negligence because Toddler started the fire, not Mom. B) Mom is not likely to be held liable in negligence to Friend because Friend assumed the risk by his ownership of the cabin. C) Mom is likely to be held liable in negligence because parents are vicariously responsible for the damage caused by their children. D) Mom is likely to be held liable in negligence to Friend because Mom failed to act as the reasonable prudent person when she left Toddler alone for an hour next to a kerosene lamp.

D) Mom is likely to be held liable in negligence to Friend because Mom failed to act as the reasonable prudent person when she left Toddler alone for an hour next to a kerosene lamp

About two years ago, Client was a patient of Ophthalmologist. Client alleges that during the time that he was Ophthalmologist's patient, Ophthalmologist improperly performed LASIK surgery on Client's eyes. The LASIK surgery was supposed to correct Client's poor vision. Instead, the surgery was performed skillfully, but resulted in blindness in both eyes, a risk about which Client was never informed and a reasonable person would not have known. If Client had known of the risk of blindness, he would not have opted for the surgery. Client hires Attorney Matthew to sue Ophthalmologist for medical malpractice. Regarding the underlying medical malpractice action, which of the following statements concerning the Ophthalmologist's liability is correct? A) Ophthalmologist's failure to inform Client of the material risks and available alternatives to LASIK treatment does not constitute a breach of the standard of care in negligence because every patient should know that any eye surgery could lead to blindness B) Ophthalmologist's failure to inform Client of the material risks and available alternatives to LASIK treatment does not constitute a breach of the standard of care in negligence because the reasonably patient should know that any eye surgery could result in blindness C) Ophthalmologist cannot be subject to liability in negligence because he performed the surgery skillfully D) Ophthalmologist's failure to inform Client of the material risks and available alternatives to LASIK treatment is a breach of the standard of care in negligence

D) Ophthalmologist's failure to inform Client of the material risks and available alternatives to LASIK treatment is a breach of the standard of care in negligence

Late one night, Pete places his loaded pistol on the coffee table in the family room of his home that he shares with his wife and two young children. The next afternoon Pete's 10-year-old son, Nolan, is spending time in the family room with his friend, Buddy. Nolan picks up the gun and plays with it, accidentally causing it to discharge. Buddy is injured by the gunshot. In a negligence action against Pete brought on behalf of Buddy, can Pete be held liable for negligence? A) Pete cannot be held liable in negligence for injuries to Buddy because Pete never intended for Buddy or anyone else to be injured B) Pete cannot be held liable in negligence for injuries to Buddy because Nolan is the person who actually discharged the gun and is therefore the sole proximate cause of Buddy's injuries C) Pete may be held liable in negligence because parents are liable for injuries caused by their children D) Pete may be held liable in negligence because Pete carelessly provided Nolan access to the gun in circumstances in which it was foreseeable that Nolan might use it improperly

D) Pete may be held liable in negligence because Pete carelessly provided Nolan access to the gun in circumstances in which it was foreseeable that Nolan might use it improperly

The negligence of Defendant Driver while driving his car resulted in an accident, which set aflame the car driven by Polly Plaintiff. Plaintiff was able to get out of her car and quickly opened the door to the backseat in order to save her two-year old child who was fastened in a child's seat. Plaintiff did this even though she knew that flames surrounded the car seat and appreciated that she was likely to be burned. Miraculously, Plaintiff saved the life of her child, but, in the process, she was herself severely burned. Plaintiff sues Defendant Driver for negligently causing the accident. What is the most likely result in most jurisdictions? A) Plaintiff will lose because she assumed the risk by voluntarily proceeding to encounter a known danger B) Plaintiff will lose because she was contributorily negligent C) Plaintiff will lose because she both assumed the risk and was contributorily negligent D) Plaintiff will prevail because her decision to encounter the risk was not unreasonable

D) Plaintiff will prevail because her decision to encounter the risk was not unreasonable

Taylor Toddler, 2½ years old, was injured when she ran out into the street in front of her family's house while her father was distracted by a report on the radio that his favorite football team had just scored a winning touchdown. She was struck by a car driven by Unfortunate Uma who was driving 10 miles over the posted speed limit of 25 m.p.h. In a negligence action brought by Toddler's parents against Uma to recover damages for Taylor's injuries in a jurisdiction that follows the traditional rule of contributory negligence and the majority approach to negligence per se, what result is most likely? A) Toddler's parents will recover nothing on her behalf because driving 10 miles over a posted speed limit is not negligent B) Toddler's parents will recover nothing on her behalf because her dad's negligence will bar recovery C) Toddler's parents will recover nothing on her behalf because of Toddler's contributory negligence D) Toddler's parents will recover the damages resulting from her injury

D) Toddler's parents will recover the damages resulting from her injury

Gas Co. supplies natural gas for the county and hires Supply Inc. to install a service line to carry gas to a new neighborhood. Supply Inc.'s employees negligently cause a slight leak in one of the gas pipes. The resulting leak causes an explosion that injures Neighbor. In a lawsuit brought by Neighbor against Gas Co., may Gas Co. be subject to vicarious or direct liability? a) Gas Co. may be subject to vicarious liability for the negligence of Supply Inc. because Gas Co. hired Supply Inc. to carry out an activity that posed a highly dangerous risk and Supply Inc.'s negligence was a factual cause of Neighbor's harm. b) Gas Co. may not be subject to vicarious liability for the negligence of Supply Inc. because Supply Inc. was an independent contractor. c) Gas Co. may be subject to direct liability for the negligence of Supply Inc. because Gas Co. hired Supply Inc. and Supply Inc.'s negligence was a factual harm of Neighbor's harm. d) Gas Co. may be subject to direct liability for the negligence of Supply Co. because Gas Co. hired Supply Co. to carry out a highly dangerous activity.

a) Gas Co. may be subject to vicarious liability for the negligence of Supply Inc. because Gas Co. hired Supply Inc. to carry out an activity that posed a highly dangerous risk and Supply Inc.'s negligence was a factual cause of Neighbor's harm

The primary basis of the court's holding in Ives v South Buffalo Railway Co. is that each party in a tort case, as a matter of due process, is entitled to: a) have his case decided under the fundamental principles of law in place at the time of the adoption of the state and federal constitutions. These fundamental principles include that no defendant shall be held liable unless he is shown to have acted with fault b) be present before the decision-making body which decides whether compensation for her or his workplace injury is awarded c) have his case decided under the common law rather than statutory law d) receive full compensation for his injuries

a) have his case decided under the fundamental principles of law in place at the time of the adoption of the state and federal constitutions. These fundamental principles include that no defendant shall be held liable unless he is shown to have acted with fault

Hiker gets lost in the woods. As hiker is trying to find his way out of the woods, he is attacked by a bear. As the hiker is running from the attacking bear, he sees a cabin in the woods. The door of the cabin has a sign "Keep Out". In desperation after reading the sign, the hiker tries the door and its unlocked. He enters the cabin and shuts the door behind him. About twenty minutes later the hiker is rescued and leaves the cabin unharmed exactly as he found it. Owner of the cabin is furious that Hiker entered the cabin without permission and sues the hiker for trespass to land. Is the hiker liable for trespass? a) No, hiker is not liable for trespass to land because he was exercising the privilege of private necessity b) Yes, hiker is liable for trespass to land because he intentionally entered the property in the possession of another without consent c) Yes, trespass to land is actionable on nominal damages only and so the fact that the hiker did no harm to the cabin is immaterial d) No, because a person may freely enter anyone's property as long as the entry will reasonably benefit the person

a) no, hiker is not liable for trespass to land because he was exercising the privilege of private necessity

Which of the following is NOT a goal or objective of proponents of a workers' compensation system: a) punishing the company responsible for the worker's injury b) reducing the transaction costs involved in compensating victims for their injuries c) loss distribution, that is, placing the liability on the class of actors in the best position to distribute the losses d) loss minimization, that is, encouraging companies to make workplaces safer by holding them liable for a much higher percentage of workplace injuries than is the case under the common law

a) punishing the company responsible for the worker's injury

Which of the following was not a factor leading American courts in the mid-nineteenth century to begin to require fault on the defendant's part to establish liability? a) the common law in place at the time of the adoption of the state and federal constitutions b) a desire to avoid imposing crippling liability costs on newly emerging railroads and other industrial enterprises c) An increasing focus on moral fault in American Society d) A belief that "no liability without fault" was in accord with contemporary political liberalism because it allowed people to act freely without imposing liability costs

a) the common law in place at the time of the adoption of the state and federal constitutions

The State of Merlin, an urban state, passed a statute that comprehensively regulates noise levels. A new nightclub recently opened for business. The state licensed the night club and, in the process, approved the level of sound emanating from it. A neighbor who values serenity sued the club, seeking both injunctive relief and damages. What is the effect of the statute on the legal action? a) The statute does not affect the action because the traditional rule is that just because an actor complies with all statutes, this does not necessarily mean that the actor is not liable in tort. b) The statute does not affect the action because only a federal statute can preempt a state common law tort action. c) The statute is admissible and the jury should consider it as a factor in deciding whether the club is liable for nuisance. d) Because the statute comprehensively regulates noise, it displaces the nuisance action and the judge should rule for the club as a matter of law.

d) Because the statute comprehensively regulates noise, it displaces the nuisance action and the judge should rule for the club as a matter of law

Electric Company has strung power lines along the bank of local river approximately 20 feet above the water's level. Recreational boaters, with masts sometimes exceeding the power lines by no more than one foot, often sail up and down the river. Polly is a guest on the boat of Friend, who is unfamiliar with this particular segment of the river. The height of the mast of friend's sailboat reaches 21 feet above the water line. As the boat approaches the shore, Polly (who does not see the power line) is holding the mast when it comes into contact with the power line above. Due to the contact, Polly suffers severe electrical burns. In Polly's negligence action against Electric Company, the evidence shows that the likelihood of contact between sailboat masts and the power lines in this segment of the river is considerable, the severity of injuries when such contacts occur will probably be extreme, and the cost of raising the height of power lines another 3-4 feet higher would be moderate. Can Electric company be held liable in negligence? a) Electric Company can be held liable in negligence to Polly for failing to raise the height of its power lines because Electric Company installed the power lines b) Electric Company can be held liable in negligence to Polly for failing to raise the height of its power lines because Electric Company's cost of raising the power lines 3-4 feet higher is moderate as weighed against the considerable likelihood of contact between a mast and the lines and the severity of probable potential injuries c) Electric Company cannot be held liable in negligence to Polly for her injuries because her injuries were caused by Friend's negligence, not Electric Company's negligence d) Electric Company cannot be held liable in negligence to Polly because it was Friend's sailboat, not Polly's sailboat

b) Electric Company can be held liable in negligence to Polly for failing to raise the height of its power lines because Electric Company's cost of raising the power lines 3-4 feet higher is moderate as weighed against the considerable likelihood of contact between a mast and the lines and the severity of probable potential injuries

Hotel Owner contacts Pipe Co. to deliver and install a large pipe to display a sign for its hotel. Hotel Owner thoroughly vets Pipe Co., and Pipe Co's representative sends Hotel Owner information showing that Pipe Co. has extensive experience in installing pipes like this and that Pipe Co. is bonded for this purpose. When Pipe Co's workers attempt the installation by using their truck, hooks, chains and additional poles for stabilization, the pipe falls, bouncing on a passing car and injuring Driver. In a lawsuit brought by Driver against Hotel Owner for injuries sustained when the pipe fell on his car, may Hotel Owner be subject to direct or vicarious liability? a) Hotel Owner may be subject to direct liability in negligence because he failed to use reasonable care in hiring Pipe Co. b) Hotel owner may not be subject to direct liability for the actions of Pipe Co because Hotel Owner did not owe a duty of care with respect to the manner in which Pipe Co performed its work and did not retain control over any part of its work. c) Hotel Owner may be subject to vicarious liability in negligence because he hired Pipe Co and Pipe Co's negligence factually caused injuries to Driver d) Hotel Owner may be subject to vicarious liability for Driver's injuries because employers are responsible for the torts of their independent contractors

b) Hotel owner may not be subject to direct liability for the actions of Pipe Co because Hotel Owner did not owe a duty of care with respect to the manner in which Pipe Co performed its work and did not retain control over any part of its work

Soda Company, a company that provides beverages to local businesses, employs Delivery Person and has provided Delivery Person with detailed instructions on exactly how to conduct its business through the workday. Delivery Person is delivering beverages to SuperStore. Teenager is shopping at SuperStore, when, while hauling sodas into SuperStore, Delivery Person carelessly runs over Teenager's food, spraining and bruising it. In a negligence action brought against Soda Company on Teenager's behalf for the injuries sustained when Delivery Person ran over his foot, may Soda Company be held vicariously liable? a) Soda company may not be held vicariously liable for Delivery Person's negligence because Delivery Person was an independent contractor at the time of the accident b) Soda company may be held vicariously liable for Delivery Person's negligence because Delivery Person's was an employee at the time of the accident with teenage c) Soda company may not be held vicariously liable for Delivery Person's negligence if the detailed instructions given to Delivery Person made clear Delivery Person was not to run over anyone's food while on a delivery d) Soda Company may be held vicariously liable for Delivery Person's negligence if Delivery Person was on a frolic

b) Soda Company may be held vicariously liable for Delivery Person's negligence because Delivery Person was an employee at the time of the accident with teenager

Smog, Inc., the manufacturer of unique, advanced laboratory equipment used for cancer research, recently built a new manufacturing plant in a neighborhood sparsely populated except for a single landscape artist, Adrian Artist. Smog produces considerable air pollutants that cannot be prevented. Artist sues to enjoin the operation of the plant as a private nuisance. Which of the following results is most likely under the majority view as expressed in the Restatement (Second) of Torts? a) The trial court will enjoin the continued operation of the plant because the plant is interfering with the use and enjoyment of Artist's land and the harm cannot be adequately compensated with damages. b) The trial court will not enjoin the operation of Smog's plant because the weighing of the social costs and benefits of its operations tip in favor of Smog. c) Artist will not prevail because she cannot prove that Smog was negligent, and liability for private nuisance requires that a defendant be negligent. d) Artist will not prevail because Smog's operations violate no laws.

b) The trial court will not enjoin the operation of Smog's plant because the weighing of the social costs and benefits of its operations tip in favor of Smog

The authority of common law judges rests primarily on the fact that they: a) are always elected by the people b) begin their decision-making process by reasoning from precedents c) take an oath to be fair and impartial d) can have their judgements in any particular case superseded by statutes passed by the legislature

b) begin their decision-making process by reasoning from precedents

The most important way that the writ of trespass on the case was distinguished from the writ of trespass was: a) the plaintiff could recover without showing fault under the writ of trespass on the case, but not when proceeding under the writ of trespass b) the plaintiff recovered for consequential, but not immediate, harms c) the defendant's actions were voluntary instead of involuntary d) the defendant could prevail under a defense of general traverse, but not special traverse

b) the plaintiff recovered for consequential, but not immediate, harms

Blasting Co. is in the business of razing buildings for districts that are building up their communities. Blasting Co. is aware that there are dangers inherent in using dynamite to raze buildings, and although it adopts all reasonable precautions, it is likely that there will be some unintended damages caused by way of flying debris or the concussion of the blast. Blasting Co. has been employed by City to raze several buildings near downtown. Several miles away, Kylie has been farming minks for their fur. Unfortunately, mink are very sensitive and have very nervous dispositions, and after feeling the vibration of the blasting from many miles away (which vibrations were not detectable to humans), several of Kylie's female minks that had just given birth ate their offspring. Kylie sues Blasting Co. in strict liability for the loss of the baby minks. Should Blasting Co. be subject to strict liability? a) Blasting Co. should be subject to strict liability for the injuries to Kylie's mink farm because Blasting Co. was engaged in an abnormally dangerous activity. b) Blasting Co. should be liable in negligence to Kylie because its blasting of dynamite was a factual cause of the injuries to Kylie's mink farm. c) Blasting Co. should not subject to strict liability for the injuries to Kylie's mink farm because the destruction of mink kittens by their mothers is not the type of damage that makes blasting an abnormally dangerous activity. d) Blasting Co. should not be subject to strict liability because the blasting was not a factual cause of the destruction of the mink kittens.

c) Blasting Co. should not subject to strict liability for the injuries to Kylie's mink farm because the destruction of mink kittens by their mothers is not the type of damage that makes blasting an abnormally dangerous activity

The workers' compensation statute considered in Ives v South Buffalo Railway Co.: a) required the claimant to prove the employer's fault, but eliminated the affirmative defenses of assumption of the risk, the fellow servant rule and, in most cases, contributory negligence b) Eliminated the requirement that the claimant prove fault in order to recover, but it preserved the affirmative defenses of assumption of the risk, the fellow servant rule, and in most cases, contributory negligence c) Eliminated both the requirement that the claimant prove fault in order to recover and the affirmative defenses of assumption of the risk, the fellow servant rule and, in most cases, contributory negligence d) Enabled claimants to recover damages that would compensate them for all losses they would have recovered in a common law tort action

c) Eliminated both the requirement that the claimant prove fault in order to recover and the affirmative defenses of assumption of the risk, the fellow servant rule, and in most cases, contributory negligence

Farmer initiates a controlled burn fire to dispose of dry straw spread over his 50 acres of land. For fires of this sort, there are appropriate precautions that must be taken, such as placing various types of obstacles at the property's boundary line. However, even when all reasonable precautions are taken, such fires escape Farmer's property 10% of the time. Because of the size of these fires, when a fire escapes the damage done to neighboring property is likely to be substantial. Farmer takes all the reasonable precautions to ensure that the fire will remain contained. However, when Farmer's fire is in progress, the wind unexpectedly picks up causing the fire to spread to the property immediately adjoining Farmer's land. Neighbors property is damaged as a result. In a lawsuit brought by Neighbor against Farmer for property damage caused by the fire, may Farmer be held liable? a) Farmer may be held liable in negligence for the injuries to Neighbor's property because Farmer failed to act as the reasonable prudent person in the circumstances b) Farmer may not be held liable for the injuries to Neighbor's property because the fire was spread to Neighbor's property by a force majeure or Act of G-d and was therefore unforseeable c) Farmer may be held strictly liable for the injuries to Neighbor's property because in initiating a controlled burn fire, Farmer was engaged in an abnormally dangerous activity d) Farmer may not be held liable for the injuries to Neighbor's property because even when all reasonable precautions are taken, such fires escape Farmer's property only 10% of the time

c) Farmer may be held strictly liable for the injuries to Neighbor's property because in initiating a controlled burn fire, Farmer was engaged in abnormally dangerous activity

Tourist was walking along public road in a town he was visiting. Tourist approached a fenced parcel of land. As tourist approached the fenced area he saw many signs that read, "Private Property. Keep Out." Believing no one was looking tourist jumped the fence to examine a flower he saw on the land. As soon as tourist landed on the other side of the fence, landowner yelled at tourist and tourist immediately jumped back over the fence onto the public roadway causing no harm to landowner's property. If landowner sues tourist for trespass to land will landowner win or lose? a) Landowner will win because tourist knew that the land was owned by landowner b) Landowner will lose because tourist did not spend enough time on landowner's land to be considered a trespass to land c) Landowner will win because tourist intentionally entered onto land owner's land d) Land owner will lose because tourist did not harm landowner's land

c) Landowner will win because tourist intentionally entered onto land owner's land

Hiker gets lost in the woods. As hiker is trying to find his way out of the woods, he is attacked by a bear. As the hiker is running from the attacking bear, he sees a cabin in the woods. The door of the cabin has a sign "Keep Out". In desperation after reading the sign, the hiker tries the door and its unlocked. He enters the cabin and shuts the door behind him. Hiker is trapped in the cabin for weeks before rescue arrives and the bear is scared off. During the time he was in the cabin, hiker had to destroy wooden furniture to keep a fire going in order to prevent himself from freezing to death. He also had to make a small hole in the roof to allow snow to melt and drip into the cabin for drinking water. And finally, he had to use food stored in the cabinets in order to survive. After rescue the owner of the cabin sues hiker for trespass to land after the hiker refuses to pay for the damage done to the cabin owner's property. Is the hiker liable? a) No, hiker is not liable for trespass to land because he was exercising the privilege of private necessity b) yes, hiker is liable for trespass to land because he intentionally entered the property in the possession of another without consent c) Yes, because hiker caused actual damage to the property d) No because a person may freely enter anyone's property as long as the entry will reasonably benefit the person

c) Yes, because hiker caused actual damage to the property

During the past century, when deciding torts cases, most American state supreme courts: a) always follow their own precedents b) follow precedents unless they have been superseded by state statutes c) begin with a presumption that they will follow precedents but may, in unusual cases, overrule precedents because of factors including social and economic considerations d) reason almost exclusively from fairness and economic and social policies

c) begin with a presumption that they will follow precedents, but may, in unusual cases, overrule precedents because of factors including social and economic considerations

In Ives v South Buffalo Railway Co., the Court of Appeals of New York held for the defendant because: a) a state statute can never change or modify the common law b) the workers' compensation statute violated the due process clause of the state constitution though it did not violate the Due Process Clause of the U.S. Constitution c) The workers' compensation statute violated the due process clauses of both the federal and state constitutions d) The workers' compensation statute deprived the defendant of a right to a jury trial

c) the workers' compensation statute violated the due process clauses of both the federal and the state constitutions

Blasting Co. is in the business of razing buildings for districts that are building up their communities. Blasting Co. is aware that there are dangers inherent in using dynamite to raze buildings, and although it adopts all reasonable precautions, it is likely that there will be some unintended damages caused by way of flying debris or the concussion of the blast. Blasting Co. has been employed by City to raze several buildings near downtown. Unfortunately, during this particular blast, even though Blasting Co. used all of its routine precautions, Driver's car was damaged by flying debris. Driver sues Blasting Co., alleging that it should be strictly liable for the damage to Driver's car. Should Blasting Co. be held liable? a) Blasting Co. should not be held strictly liable for injuries to Driver's car because it used all due care to prevent any damage from flying debris. b) Blasting Co. should be subject to liability in negligence because it used all due care and Driver's car was still damaged. c) Blasting Co. should not subject to strict liability because Driver suffered property damage through no fault of its own. d) Blasting Co. may be held strictly liable for the injuries to Driver's car because Blasting Co. was engaged in an abnormally dangerous activity.

d) Blasting Co. may be held strictly liable for the injuries to Driver's car because Blasting Co. was engaged in an abnormally dangerous activity

A preschool recently began operating near a military base. The most affordable and practical location for Preschool was immediately adjacent to Adult Bookstore. Preschool found that the nearby presence of Adult Bookstore posed a threat to the children attending Preschool and also discouraged parents from enrolling their children at Preschool. Preschool wrote Adult Bookstore informing the store of its concerns. When Adult Bookstore failed to respond, Preschool filed suit against Adult Bookstore for private nuisance and sought both damages and an injunction to force Adult Bookstore to move its operations. What outcome would you anticipate? a) Preschool will lose as a matter of law because it "came to the nuisance." b) Preschool will lose because Adult Bookstore (a) did not act with a purpose of harming Preschool, (b) did not operate its business negligently, and (c) does not qualify as an abnormally dangerous activity. c) Preschool will lose because Adult Bookstore did not violate any statutes, ordinances, or regulations. d) Preschool may win because a reasonable jury might find that Adult Bookstore intentionally and unreasonably interfered with Preschool's use and enjoyment of its land by continuing its operations after being told that its operations interfered with Preschool.

d) Preschool may win because a reasonable jury might find that Adult Bookstore intentionally and unreasonably interfered with Preschool's use and enjoyment of its land by continuing its operations after being told that its operations interfered with Preschool

It was recently discovered that a widely distributed prescription medication produced by a pharmaceutical manufacturer dramatically increases the risk of heart disease and constitutes a public health crisis. Valerie Victim, who took the medication for decades, has recently been diagnosed with heart disease and treated for it. Victim sues Big Pharma, Inc., the manufacturer of the drug, for both private nuisance and public nuisance. Big Pharma files a motion to dismiss both claims. What result is most likely? a) Each of the motions to dismiss will be denied, and Victim will be allowed to proceed on both claims. b) The court will dismiss the private nuisance claim because it does not involve interference with land, but will allow the public nuisance claim to proceed because the right to be healthy is a right held in common by members of the general public. c) The court will dismiss the public nuisance claim because most courts hold that manufacturers cannot be held liable under public nuisance for the manufacture and distribution of products, but will allow the private nuisance claim to proceed because Victim seeks to enforce a private right. d) The court will grant both motions to dismiss. The private nuisance claim will be dismissed because the claim does not involve interference with the use and enjoyment of land. The public nuisance claim will be dismissed because the manufacture and distribution of products does not create liability under the claim.

d) The court will grant both motions to dismiss. The private nuisance claim will be dismissed because the claim does not involve interference with the use and enjoyment of land. The public nuisance claim will be dismissed because the manufacture and distribution of products does not create liability under the claim

Private boater is out on the water for a leisure day when the weather radio announces the development of a sudden but strong storm in the immediate vicinity of the boater. The weather radio says that all boats in the area must seek immediate cover and be fastened to a dock, if possible. Hearing the warning, private boater ties his boat up to the nearest dock. The dock is owned by landowner who sees the boat tie up and decides he does not want the boat tied to HER dock. Landowner unties private boat from his dock just as the storm approaches, private boat drifts off into the storm colliding with other boats causing damage to other boats. Owners of other boats sue private landowner for damages caused by private boat he refused to let stay on his dock. Is landowner liable for damage? a) No, landowner is not liable for damage because she has the absolute right to exclude others from his property b) yes, landowner is liable only if she acted negligently c) no, landowner is not liable because private boater did not ask permission to use dock d) yes, landowner is liable because during private necessity the landowner's right to exclude people from his property yields to the person claiming necessity's rights to use the property

d) Yes, landowner is liable because during private necessity the landowner's right to exclude people from his property yields to the person claiming necessity's right to use the property

Under the writ system of early English common law, each writ a) gave a royal court its jurisdictional basis b) indicated which substantive law governed a case c) provided its own processes and procedures d) all of the above

d) all of the above

The most important objective of tort law under the corrective justice perspective is: a) loss minimization b) loss distribution c) minimizing transaction costs d) requiring the tortfeasor to make the victim whole after his moral wrongdoing resulted in the injury to the victim

d) requiring the tortfeasor to make the victim whole after his moral wrongdoing resulted in the injury to the victim

"Distinguishing a precedent" means showing that: a) a later decision overruled the precedent b) the legislature passed a statute superseding the precedent c) the precedent conflicted with the due process clause d) the facts of the case now before the court differ in a legally significant way from those of the precedent

d) the facts of the case now before the court differ in a legally significant way from those of the precedent

Under English Common Law in the centuries preceding American Independence: a) a requirement of no liability without fault was a fundamental principle b) the plaintiff was required to prove defendant's fault in personal injury cases, but not in cases involving land disputes c) the plaintiff was required to prove defendant's fault, but not in the absence of contributory negligence or assumption of risk d) the plaintiff was not required to prove fault. The dominant view among legal historians is that this was true for both trespass and trespass on the case. Other legal historians believe this was true only for trespass.

d) the plaintiff was not required to prove fault. The dominant view among legal historians is that this was true for both trespass and trespass on the case. Other legal historians believe this was true only for trespass


Set pelajaran terkait

NCLEX - Management of Care, Interprofessional Collaboration, Delegating & Supervising

View Set

Intro to Data Analytics Study Guide

View Set

Malcolm X, Nation of Islam and Black Power

View Set

LS23L Lab C Week 3: b-Galactosidase Assay

View Set